How to evaluate this nonelementary integral?











up vote
9
down vote

favorite
1












Let $x>0$. I have to prove that



$$
int_{0}^{infty}frac{cos x}{x^p}dx=frac{pi}{2Gamma(p)cos(pfrac{pi}{2})}tag{1}
$$



by converting the integral on the left side to a double integral using the expression below:



$$
frac{1}{x^p}=frac{1}{Gamma(p)}int_{0}^{infty}e^{-xt}t^{p-1}dttag{2}
$$



By plugging $(2)$ into $(1)$ I get the following double integral:



$$
frac{1}{Gamma(p)}int_{0}^{infty}int_{0}^{infty}e^{-xt}t^{p-1}cos xdtdxtag{3}
$$



However, I unable to proceed any further as I am unclear as to what method should I use in order to compute this integral. I thought that an appropriate change of variables could transform it into a product of two gamma functions but I cannot see how that would work. Any help would be greatly appreciated.










share|cite|improve this question




















  • 1




    Change the order of integration, then use any method from Closed form for $ int_0^infty {frac{{{x^n}}}{{1 + {x^m}}}dx }$
    – projectilemotion
    Nov 24 at 21:36

















up vote
9
down vote

favorite
1












Let $x>0$. I have to prove that



$$
int_{0}^{infty}frac{cos x}{x^p}dx=frac{pi}{2Gamma(p)cos(pfrac{pi}{2})}tag{1}
$$



by converting the integral on the left side to a double integral using the expression below:



$$
frac{1}{x^p}=frac{1}{Gamma(p)}int_{0}^{infty}e^{-xt}t^{p-1}dttag{2}
$$



By plugging $(2)$ into $(1)$ I get the following double integral:



$$
frac{1}{Gamma(p)}int_{0}^{infty}int_{0}^{infty}e^{-xt}t^{p-1}cos xdtdxtag{3}
$$



However, I unable to proceed any further as I am unclear as to what method should I use in order to compute this integral. I thought that an appropriate change of variables could transform it into a product of two gamma functions but I cannot see how that would work. Any help would be greatly appreciated.










share|cite|improve this question




















  • 1




    Change the order of integration, then use any method from Closed form for $ int_0^infty {frac{{{x^n}}}{{1 + {x^m}}}dx }$
    – projectilemotion
    Nov 24 at 21:36















up vote
9
down vote

favorite
1









up vote
9
down vote

favorite
1






1





Let $x>0$. I have to prove that



$$
int_{0}^{infty}frac{cos x}{x^p}dx=frac{pi}{2Gamma(p)cos(pfrac{pi}{2})}tag{1}
$$



by converting the integral on the left side to a double integral using the expression below:



$$
frac{1}{x^p}=frac{1}{Gamma(p)}int_{0}^{infty}e^{-xt}t^{p-1}dttag{2}
$$



By plugging $(2)$ into $(1)$ I get the following double integral:



$$
frac{1}{Gamma(p)}int_{0}^{infty}int_{0}^{infty}e^{-xt}t^{p-1}cos xdtdxtag{3}
$$



However, I unable to proceed any further as I am unclear as to what method should I use in order to compute this integral. I thought that an appropriate change of variables could transform it into a product of two gamma functions but I cannot see how that would work. Any help would be greatly appreciated.










share|cite|improve this question















Let $x>0$. I have to prove that



$$
int_{0}^{infty}frac{cos x}{x^p}dx=frac{pi}{2Gamma(p)cos(pfrac{pi}{2})}tag{1}
$$



by converting the integral on the left side to a double integral using the expression below:



$$
frac{1}{x^p}=frac{1}{Gamma(p)}int_{0}^{infty}e^{-xt}t^{p-1}dttag{2}
$$



By plugging $(2)$ into $(1)$ I get the following double integral:



$$
frac{1}{Gamma(p)}int_{0}^{infty}int_{0}^{infty}e^{-xt}t^{p-1}cos xdtdxtag{3}
$$



However, I unable to proceed any further as I am unclear as to what method should I use in order to compute this integral. I thought that an appropriate change of variables could transform it into a product of two gamma functions but I cannot see how that would work. Any help would be greatly appreciated.







multivariable-calculus gamma-function






share|cite|improve this question















share|cite|improve this question













share|cite|improve this question




share|cite|improve this question








edited Nov 24 at 21:15









Key Flex

7,03931229




7,03931229










asked Nov 24 at 21:14









Phillip

553




553








  • 1




    Change the order of integration, then use any method from Closed form for $ int_0^infty {frac{{{x^n}}}{{1 + {x^m}}}dx }$
    – projectilemotion
    Nov 24 at 21:36
















  • 1




    Change the order of integration, then use any method from Closed form for $ int_0^infty {frac{{{x^n}}}{{1 + {x^m}}}dx }$
    – projectilemotion
    Nov 24 at 21:36










1




1




Change the order of integration, then use any method from Closed form for $ int_0^infty {frac{{{x^n}}}{{1 + {x^m}}}dx }$
– projectilemotion
Nov 24 at 21:36






Change the order of integration, then use any method from Closed form for $ int_0^infty {frac{{{x^n}}}{{1 + {x^m}}}dx }$
– projectilemotion
Nov 24 at 21:36












3 Answers
3






active

oldest

votes

















up vote
9
down vote



accepted










The Laplace transform of $cos x$ is $frac{s}{1+s^2}$ and the inverse Laplace transform of $frac{1}{x^p}$ is $frac{s^{p-1}}{Gamma(p)}$, hence
$$ int_{0}^{+infty}frac{cos x}{x^p},dx = frac{1}{Gamma(p)}int_{0}^{+infty}frac{s^p}{s^2+1},ds=frac{1}{Gamma(p)}int_{0}^{pi/2}left(tan uright)^p,du $$
equals
$$ begin{eqnarray*}frac{1}{Gamma(p)}int_{0}^{1} v^p (1-v^2)^{-(p+1)/2},dv&=&frac{1}{2,Gamma(p)}int_{0}^{1}w^{(p-1)/2}(1-w)^{-(p+1)/2},dw\& =& frac{Bleft(tfrac{1+p}{2},tfrac{1-p}{2}right)}{2,Gamma(p)}end{eqnarray*} $$
or
$$ frac{Gammaleft(frac{1+p}{2}right)Gammaleft(frac{1-p}{2}right)}{2,Gamma(p)}= frac{pi}{2,Gamma(p)sinleft(frac{pi}{2}(p+1)right)}=frac{pi}{2,Gamma(p)cosleft(frac{pi p}{2}right)}$$
as wanted. We have exploited the Beta function and the reflection formula for the $Gamma$ function.






share|cite|improve this answer




























    up vote
    5
    down vote













    Your given integral is closely related to the Mellin transform and can be evaluated by using Ramanujan's Master Theorem.




    Ramanujans Master Theorem



    Let $f(x)$ be an analytic function with a series representation of the form
    $$f(x)=sum_{k=0}^{infty}frac{phi(k)}{k!}(-x)^k$$then the Mellin Transform of this function is given by
    $$int_0^{infty}x^{s-1}f(x)dx=Gamma(s)phi(-s)$$




    Therefore expand the cosine function as Taylor series expansion to get



    $$begin{align}
    mathfrak{I}=int_0^{infty}cos(x)x^{-p}dx&=int_0^{infty}x^{-p}sum_{n=0}^{infty}(-1)^nfrac{x^{2n}}{(2n)!}dx
    end{align}$$



    In order to bring the above integral in the wanted form for the usage of Ramanujan's Master Theorem apply the substitution $x^2=u$. So we further get



    $$begin{align}
    mathfrak{I}=int_0^{infty}x^{-p}sum_{n=0}^{infty}(-1)^nfrac{x^{2n}}{(2n)!}dx&=int_0^{infty}x^{-p}sum_{n=0}^{infty}frac{1}{(2n)!}(-x^2)^ndx\
    &=int_0^{infty}u^{-p/2}sum_{n=0}^{infty}frac{1}{(2n)!}(-u)^nfrac{du}{2sqrt{u}}\
    &=frac12int_0^{infty}u^{-(p+1)/2}sum_{n=0}^{infty}frac{1}{(2n)!}(-u)^ndu\
    &=frac12int_0^{infty}u^{-(p+1)/2}sum_{n=0}^{infty}frac{n!/(2n)!}{n!}(-u)^ndu
    end{align}$$



    By using the relation $Gamma(n)=(n-1)!$ which is valid for all $ninmathbb N$ we can consider the last integral as an application of Ramanujan's Master Theorem with $s=-frac{p-1}2$ and $phi(n)=frac{Gamma(n+1)}{Gamma(2n+1)}$. By finally using the Theorem we obtain



    $$begin{align}
    mathfrak{I}=frac12int_0^{infty}u^{-(p+1)/2}sum_{n=0}^{infty}frac{n!/(2n)!}{n!}(-u)^ndu&=frac12Gammaleft(-frac{p-1}2right)frac{Gammaleft(frac{p-1}2+1right)}{Gammaleft(2left(frac{p-1}2right)+1right)}\
    &=frac1{2Gamma(p)}Gammaleft(1+frac{p-1}2right)Gammaleft(-frac{p-1}2right)
    end{align}$$



    Now by applying Euler's Reflection Formula with $z=1+frac{p-1}2$ we moreover get



    $$begin{align}
    mathfrak{I}=frac1{2Gamma(p)}Gammaleft(1+frac{p-1}2right)Gammaleft(-frac{p-1}2right)&=frac1{2Gamma(p)}frac{pi}{sinleft(pileft(1+frac{p-1}2right)right)}\
    &=frac1{2Gamma(p)}frac{pi}{sinleft(frac{ppi}2+frac{pi}2right)}\
    &=frac1{2Gamma(p)}frac{pi}{cosleft(frac{ppi}2right)}
    end{align}$$



    where within the last step the fundamental relation $sinleft(x+frac{pi}2right)=cos(x)$ was used. Thus for the original integral $mathfrak{I}$ we get




    $$mathfrak{I}=int_0^{infty}cos(x)x^{-p}dx=frac{pi}{2Gamma(p)cosleft(pfrac{pi}2right)}$$







    share|cite|improve this answer






























      up vote
      3
      down vote













      Hint: $displaystyleint_{0}^{infty}frac{cos x}{x^p}dx= text{Real part of}:int_{0}^{infty}frac{e^{iz}}{z^p}dz$ and use residue theorem. This has a pole of order $p$ hence the term $Gamma (p)$ in the denominator.






      share|cite|improve this answer





















      • This only works for integral $p$, right?
        – AccidentalFourierTransform
        Nov 24 at 22:13










      • @AccidentalFourierTransform: Your comment makes me wonder whether there is a way to bootstrap from integer $p$ to the complex $p$...
        – user21820
        Nov 25 at 4:13










      • @user21820 Something in the spirit of the Identity theorem, or more generally Analytic continuation, I guess...
        – AccidentalFourierTransform
        Nov 25 at 4:16










      • @AccidentalFourierTransform: But here the answer is a meromorphic function of $p$, and we don't even have an accumulation point. So do you have a real proof? =)
        – user21820
        Nov 25 at 4:22










      • @user21820 Oh, no, not at all. I was just pointing out that Yidati's answer was only valid for integer $p$. I didn't want to suggest that I have a better answer.
        – AccidentalFourierTransform
        Nov 25 at 4:26











      Your Answer





      StackExchange.ifUsing("editor", function () {
      return StackExchange.using("mathjaxEditing", function () {
      StackExchange.MarkdownEditor.creationCallbacks.add(function (editor, postfix) {
      StackExchange.mathjaxEditing.prepareWmdForMathJax(editor, postfix, [["$", "$"], ["\\(","\\)"]]);
      });
      });
      }, "mathjax-editing");

      StackExchange.ready(function() {
      var channelOptions = {
      tags: "".split(" "),
      id: "69"
      };
      initTagRenderer("".split(" "), "".split(" "), channelOptions);

      StackExchange.using("externalEditor", function() {
      // Have to fire editor after snippets, if snippets enabled
      if (StackExchange.settings.snippets.snippetsEnabled) {
      StackExchange.using("snippets", function() {
      createEditor();
      });
      }
      else {
      createEditor();
      }
      });

      function createEditor() {
      StackExchange.prepareEditor({
      heartbeatType: 'answer',
      convertImagesToLinks: true,
      noModals: true,
      showLowRepImageUploadWarning: true,
      reputationToPostImages: 10,
      bindNavPrevention: true,
      postfix: "",
      imageUploader: {
      brandingHtml: "Powered by u003ca class="icon-imgur-white" href="https://imgur.com/"u003eu003c/au003e",
      contentPolicyHtml: "User contributions licensed under u003ca href="https://creativecommons.org/licenses/by-sa/3.0/"u003ecc by-sa 3.0 with attribution requiredu003c/au003e u003ca href="https://stackoverflow.com/legal/content-policy"u003e(content policy)u003c/au003e",
      allowUrls: true
      },
      noCode: true, onDemand: true,
      discardSelector: ".discard-answer"
      ,immediatelyShowMarkdownHelp:true
      });


      }
      });














      draft saved

      draft discarded


















      StackExchange.ready(
      function () {
      StackExchange.openid.initPostLogin('.new-post-login', 'https%3a%2f%2fmath.stackexchange.com%2fquestions%2f3012090%2fhow-to-evaluate-this-nonelementary-integral%23new-answer', 'question_page');
      }
      );

      Post as a guest















      Required, but never shown

























      3 Answers
      3






      active

      oldest

      votes








      3 Answers
      3






      active

      oldest

      votes









      active

      oldest

      votes






      active

      oldest

      votes








      up vote
      9
      down vote



      accepted










      The Laplace transform of $cos x$ is $frac{s}{1+s^2}$ and the inverse Laplace transform of $frac{1}{x^p}$ is $frac{s^{p-1}}{Gamma(p)}$, hence
      $$ int_{0}^{+infty}frac{cos x}{x^p},dx = frac{1}{Gamma(p)}int_{0}^{+infty}frac{s^p}{s^2+1},ds=frac{1}{Gamma(p)}int_{0}^{pi/2}left(tan uright)^p,du $$
      equals
      $$ begin{eqnarray*}frac{1}{Gamma(p)}int_{0}^{1} v^p (1-v^2)^{-(p+1)/2},dv&=&frac{1}{2,Gamma(p)}int_{0}^{1}w^{(p-1)/2}(1-w)^{-(p+1)/2},dw\& =& frac{Bleft(tfrac{1+p}{2},tfrac{1-p}{2}right)}{2,Gamma(p)}end{eqnarray*} $$
      or
      $$ frac{Gammaleft(frac{1+p}{2}right)Gammaleft(frac{1-p}{2}right)}{2,Gamma(p)}= frac{pi}{2,Gamma(p)sinleft(frac{pi}{2}(p+1)right)}=frac{pi}{2,Gamma(p)cosleft(frac{pi p}{2}right)}$$
      as wanted. We have exploited the Beta function and the reflection formula for the $Gamma$ function.






      share|cite|improve this answer

























        up vote
        9
        down vote



        accepted










        The Laplace transform of $cos x$ is $frac{s}{1+s^2}$ and the inverse Laplace transform of $frac{1}{x^p}$ is $frac{s^{p-1}}{Gamma(p)}$, hence
        $$ int_{0}^{+infty}frac{cos x}{x^p},dx = frac{1}{Gamma(p)}int_{0}^{+infty}frac{s^p}{s^2+1},ds=frac{1}{Gamma(p)}int_{0}^{pi/2}left(tan uright)^p,du $$
        equals
        $$ begin{eqnarray*}frac{1}{Gamma(p)}int_{0}^{1} v^p (1-v^2)^{-(p+1)/2},dv&=&frac{1}{2,Gamma(p)}int_{0}^{1}w^{(p-1)/2}(1-w)^{-(p+1)/2},dw\& =& frac{Bleft(tfrac{1+p}{2},tfrac{1-p}{2}right)}{2,Gamma(p)}end{eqnarray*} $$
        or
        $$ frac{Gammaleft(frac{1+p}{2}right)Gammaleft(frac{1-p}{2}right)}{2,Gamma(p)}= frac{pi}{2,Gamma(p)sinleft(frac{pi}{2}(p+1)right)}=frac{pi}{2,Gamma(p)cosleft(frac{pi p}{2}right)}$$
        as wanted. We have exploited the Beta function and the reflection formula for the $Gamma$ function.






        share|cite|improve this answer























          up vote
          9
          down vote



          accepted







          up vote
          9
          down vote



          accepted






          The Laplace transform of $cos x$ is $frac{s}{1+s^2}$ and the inverse Laplace transform of $frac{1}{x^p}$ is $frac{s^{p-1}}{Gamma(p)}$, hence
          $$ int_{0}^{+infty}frac{cos x}{x^p},dx = frac{1}{Gamma(p)}int_{0}^{+infty}frac{s^p}{s^2+1},ds=frac{1}{Gamma(p)}int_{0}^{pi/2}left(tan uright)^p,du $$
          equals
          $$ begin{eqnarray*}frac{1}{Gamma(p)}int_{0}^{1} v^p (1-v^2)^{-(p+1)/2},dv&=&frac{1}{2,Gamma(p)}int_{0}^{1}w^{(p-1)/2}(1-w)^{-(p+1)/2},dw\& =& frac{Bleft(tfrac{1+p}{2},tfrac{1-p}{2}right)}{2,Gamma(p)}end{eqnarray*} $$
          or
          $$ frac{Gammaleft(frac{1+p}{2}right)Gammaleft(frac{1-p}{2}right)}{2,Gamma(p)}= frac{pi}{2,Gamma(p)sinleft(frac{pi}{2}(p+1)right)}=frac{pi}{2,Gamma(p)cosleft(frac{pi p}{2}right)}$$
          as wanted. We have exploited the Beta function and the reflection formula for the $Gamma$ function.






          share|cite|improve this answer












          The Laplace transform of $cos x$ is $frac{s}{1+s^2}$ and the inverse Laplace transform of $frac{1}{x^p}$ is $frac{s^{p-1}}{Gamma(p)}$, hence
          $$ int_{0}^{+infty}frac{cos x}{x^p},dx = frac{1}{Gamma(p)}int_{0}^{+infty}frac{s^p}{s^2+1},ds=frac{1}{Gamma(p)}int_{0}^{pi/2}left(tan uright)^p,du $$
          equals
          $$ begin{eqnarray*}frac{1}{Gamma(p)}int_{0}^{1} v^p (1-v^2)^{-(p+1)/2},dv&=&frac{1}{2,Gamma(p)}int_{0}^{1}w^{(p-1)/2}(1-w)^{-(p+1)/2},dw\& =& frac{Bleft(tfrac{1+p}{2},tfrac{1-p}{2}right)}{2,Gamma(p)}end{eqnarray*} $$
          or
          $$ frac{Gammaleft(frac{1+p}{2}right)Gammaleft(frac{1-p}{2}right)}{2,Gamma(p)}= frac{pi}{2,Gamma(p)sinleft(frac{pi}{2}(p+1)right)}=frac{pi}{2,Gamma(p)cosleft(frac{pi p}{2}right)}$$
          as wanted. We have exploited the Beta function and the reflection formula for the $Gamma$ function.







          share|cite|improve this answer












          share|cite|improve this answer



          share|cite|improve this answer










          answered Nov 24 at 21:27









          Jack D'Aurizio

          283k33275653




          283k33275653






















              up vote
              5
              down vote













              Your given integral is closely related to the Mellin transform and can be evaluated by using Ramanujan's Master Theorem.




              Ramanujans Master Theorem



              Let $f(x)$ be an analytic function with a series representation of the form
              $$f(x)=sum_{k=0}^{infty}frac{phi(k)}{k!}(-x)^k$$then the Mellin Transform of this function is given by
              $$int_0^{infty}x^{s-1}f(x)dx=Gamma(s)phi(-s)$$




              Therefore expand the cosine function as Taylor series expansion to get



              $$begin{align}
              mathfrak{I}=int_0^{infty}cos(x)x^{-p}dx&=int_0^{infty}x^{-p}sum_{n=0}^{infty}(-1)^nfrac{x^{2n}}{(2n)!}dx
              end{align}$$



              In order to bring the above integral in the wanted form for the usage of Ramanujan's Master Theorem apply the substitution $x^2=u$. So we further get



              $$begin{align}
              mathfrak{I}=int_0^{infty}x^{-p}sum_{n=0}^{infty}(-1)^nfrac{x^{2n}}{(2n)!}dx&=int_0^{infty}x^{-p}sum_{n=0}^{infty}frac{1}{(2n)!}(-x^2)^ndx\
              &=int_0^{infty}u^{-p/2}sum_{n=0}^{infty}frac{1}{(2n)!}(-u)^nfrac{du}{2sqrt{u}}\
              &=frac12int_0^{infty}u^{-(p+1)/2}sum_{n=0}^{infty}frac{1}{(2n)!}(-u)^ndu\
              &=frac12int_0^{infty}u^{-(p+1)/2}sum_{n=0}^{infty}frac{n!/(2n)!}{n!}(-u)^ndu
              end{align}$$



              By using the relation $Gamma(n)=(n-1)!$ which is valid for all $ninmathbb N$ we can consider the last integral as an application of Ramanujan's Master Theorem with $s=-frac{p-1}2$ and $phi(n)=frac{Gamma(n+1)}{Gamma(2n+1)}$. By finally using the Theorem we obtain



              $$begin{align}
              mathfrak{I}=frac12int_0^{infty}u^{-(p+1)/2}sum_{n=0}^{infty}frac{n!/(2n)!}{n!}(-u)^ndu&=frac12Gammaleft(-frac{p-1}2right)frac{Gammaleft(frac{p-1}2+1right)}{Gammaleft(2left(frac{p-1}2right)+1right)}\
              &=frac1{2Gamma(p)}Gammaleft(1+frac{p-1}2right)Gammaleft(-frac{p-1}2right)
              end{align}$$



              Now by applying Euler's Reflection Formula with $z=1+frac{p-1}2$ we moreover get



              $$begin{align}
              mathfrak{I}=frac1{2Gamma(p)}Gammaleft(1+frac{p-1}2right)Gammaleft(-frac{p-1}2right)&=frac1{2Gamma(p)}frac{pi}{sinleft(pileft(1+frac{p-1}2right)right)}\
              &=frac1{2Gamma(p)}frac{pi}{sinleft(frac{ppi}2+frac{pi}2right)}\
              &=frac1{2Gamma(p)}frac{pi}{cosleft(frac{ppi}2right)}
              end{align}$$



              where within the last step the fundamental relation $sinleft(x+frac{pi}2right)=cos(x)$ was used. Thus for the original integral $mathfrak{I}$ we get




              $$mathfrak{I}=int_0^{infty}cos(x)x^{-p}dx=frac{pi}{2Gamma(p)cosleft(pfrac{pi}2right)}$$







              share|cite|improve this answer



























                up vote
                5
                down vote













                Your given integral is closely related to the Mellin transform and can be evaluated by using Ramanujan's Master Theorem.




                Ramanujans Master Theorem



                Let $f(x)$ be an analytic function with a series representation of the form
                $$f(x)=sum_{k=0}^{infty}frac{phi(k)}{k!}(-x)^k$$then the Mellin Transform of this function is given by
                $$int_0^{infty}x^{s-1}f(x)dx=Gamma(s)phi(-s)$$




                Therefore expand the cosine function as Taylor series expansion to get



                $$begin{align}
                mathfrak{I}=int_0^{infty}cos(x)x^{-p}dx&=int_0^{infty}x^{-p}sum_{n=0}^{infty}(-1)^nfrac{x^{2n}}{(2n)!}dx
                end{align}$$



                In order to bring the above integral in the wanted form for the usage of Ramanujan's Master Theorem apply the substitution $x^2=u$. So we further get



                $$begin{align}
                mathfrak{I}=int_0^{infty}x^{-p}sum_{n=0}^{infty}(-1)^nfrac{x^{2n}}{(2n)!}dx&=int_0^{infty}x^{-p}sum_{n=0}^{infty}frac{1}{(2n)!}(-x^2)^ndx\
                &=int_0^{infty}u^{-p/2}sum_{n=0}^{infty}frac{1}{(2n)!}(-u)^nfrac{du}{2sqrt{u}}\
                &=frac12int_0^{infty}u^{-(p+1)/2}sum_{n=0}^{infty}frac{1}{(2n)!}(-u)^ndu\
                &=frac12int_0^{infty}u^{-(p+1)/2}sum_{n=0}^{infty}frac{n!/(2n)!}{n!}(-u)^ndu
                end{align}$$



                By using the relation $Gamma(n)=(n-1)!$ which is valid for all $ninmathbb N$ we can consider the last integral as an application of Ramanujan's Master Theorem with $s=-frac{p-1}2$ and $phi(n)=frac{Gamma(n+1)}{Gamma(2n+1)}$. By finally using the Theorem we obtain



                $$begin{align}
                mathfrak{I}=frac12int_0^{infty}u^{-(p+1)/2}sum_{n=0}^{infty}frac{n!/(2n)!}{n!}(-u)^ndu&=frac12Gammaleft(-frac{p-1}2right)frac{Gammaleft(frac{p-1}2+1right)}{Gammaleft(2left(frac{p-1}2right)+1right)}\
                &=frac1{2Gamma(p)}Gammaleft(1+frac{p-1}2right)Gammaleft(-frac{p-1}2right)
                end{align}$$



                Now by applying Euler's Reflection Formula with $z=1+frac{p-1}2$ we moreover get



                $$begin{align}
                mathfrak{I}=frac1{2Gamma(p)}Gammaleft(1+frac{p-1}2right)Gammaleft(-frac{p-1}2right)&=frac1{2Gamma(p)}frac{pi}{sinleft(pileft(1+frac{p-1}2right)right)}\
                &=frac1{2Gamma(p)}frac{pi}{sinleft(frac{ppi}2+frac{pi}2right)}\
                &=frac1{2Gamma(p)}frac{pi}{cosleft(frac{ppi}2right)}
                end{align}$$



                where within the last step the fundamental relation $sinleft(x+frac{pi}2right)=cos(x)$ was used. Thus for the original integral $mathfrak{I}$ we get




                $$mathfrak{I}=int_0^{infty}cos(x)x^{-p}dx=frac{pi}{2Gamma(p)cosleft(pfrac{pi}2right)}$$







                share|cite|improve this answer

























                  up vote
                  5
                  down vote










                  up vote
                  5
                  down vote









                  Your given integral is closely related to the Mellin transform and can be evaluated by using Ramanujan's Master Theorem.




                  Ramanujans Master Theorem



                  Let $f(x)$ be an analytic function with a series representation of the form
                  $$f(x)=sum_{k=0}^{infty}frac{phi(k)}{k!}(-x)^k$$then the Mellin Transform of this function is given by
                  $$int_0^{infty}x^{s-1}f(x)dx=Gamma(s)phi(-s)$$




                  Therefore expand the cosine function as Taylor series expansion to get



                  $$begin{align}
                  mathfrak{I}=int_0^{infty}cos(x)x^{-p}dx&=int_0^{infty}x^{-p}sum_{n=0}^{infty}(-1)^nfrac{x^{2n}}{(2n)!}dx
                  end{align}$$



                  In order to bring the above integral in the wanted form for the usage of Ramanujan's Master Theorem apply the substitution $x^2=u$. So we further get



                  $$begin{align}
                  mathfrak{I}=int_0^{infty}x^{-p}sum_{n=0}^{infty}(-1)^nfrac{x^{2n}}{(2n)!}dx&=int_0^{infty}x^{-p}sum_{n=0}^{infty}frac{1}{(2n)!}(-x^2)^ndx\
                  &=int_0^{infty}u^{-p/2}sum_{n=0}^{infty}frac{1}{(2n)!}(-u)^nfrac{du}{2sqrt{u}}\
                  &=frac12int_0^{infty}u^{-(p+1)/2}sum_{n=0}^{infty}frac{1}{(2n)!}(-u)^ndu\
                  &=frac12int_0^{infty}u^{-(p+1)/2}sum_{n=0}^{infty}frac{n!/(2n)!}{n!}(-u)^ndu
                  end{align}$$



                  By using the relation $Gamma(n)=(n-1)!$ which is valid for all $ninmathbb N$ we can consider the last integral as an application of Ramanujan's Master Theorem with $s=-frac{p-1}2$ and $phi(n)=frac{Gamma(n+1)}{Gamma(2n+1)}$. By finally using the Theorem we obtain



                  $$begin{align}
                  mathfrak{I}=frac12int_0^{infty}u^{-(p+1)/2}sum_{n=0}^{infty}frac{n!/(2n)!}{n!}(-u)^ndu&=frac12Gammaleft(-frac{p-1}2right)frac{Gammaleft(frac{p-1}2+1right)}{Gammaleft(2left(frac{p-1}2right)+1right)}\
                  &=frac1{2Gamma(p)}Gammaleft(1+frac{p-1}2right)Gammaleft(-frac{p-1}2right)
                  end{align}$$



                  Now by applying Euler's Reflection Formula with $z=1+frac{p-1}2$ we moreover get



                  $$begin{align}
                  mathfrak{I}=frac1{2Gamma(p)}Gammaleft(1+frac{p-1}2right)Gammaleft(-frac{p-1}2right)&=frac1{2Gamma(p)}frac{pi}{sinleft(pileft(1+frac{p-1}2right)right)}\
                  &=frac1{2Gamma(p)}frac{pi}{sinleft(frac{ppi}2+frac{pi}2right)}\
                  &=frac1{2Gamma(p)}frac{pi}{cosleft(frac{ppi}2right)}
                  end{align}$$



                  where within the last step the fundamental relation $sinleft(x+frac{pi}2right)=cos(x)$ was used. Thus for the original integral $mathfrak{I}$ we get




                  $$mathfrak{I}=int_0^{infty}cos(x)x^{-p}dx=frac{pi}{2Gamma(p)cosleft(pfrac{pi}2right)}$$







                  share|cite|improve this answer














                  Your given integral is closely related to the Mellin transform and can be evaluated by using Ramanujan's Master Theorem.




                  Ramanujans Master Theorem



                  Let $f(x)$ be an analytic function with a series representation of the form
                  $$f(x)=sum_{k=0}^{infty}frac{phi(k)}{k!}(-x)^k$$then the Mellin Transform of this function is given by
                  $$int_0^{infty}x^{s-1}f(x)dx=Gamma(s)phi(-s)$$




                  Therefore expand the cosine function as Taylor series expansion to get



                  $$begin{align}
                  mathfrak{I}=int_0^{infty}cos(x)x^{-p}dx&=int_0^{infty}x^{-p}sum_{n=0}^{infty}(-1)^nfrac{x^{2n}}{(2n)!}dx
                  end{align}$$



                  In order to bring the above integral in the wanted form for the usage of Ramanujan's Master Theorem apply the substitution $x^2=u$. So we further get



                  $$begin{align}
                  mathfrak{I}=int_0^{infty}x^{-p}sum_{n=0}^{infty}(-1)^nfrac{x^{2n}}{(2n)!}dx&=int_0^{infty}x^{-p}sum_{n=0}^{infty}frac{1}{(2n)!}(-x^2)^ndx\
                  &=int_0^{infty}u^{-p/2}sum_{n=0}^{infty}frac{1}{(2n)!}(-u)^nfrac{du}{2sqrt{u}}\
                  &=frac12int_0^{infty}u^{-(p+1)/2}sum_{n=0}^{infty}frac{1}{(2n)!}(-u)^ndu\
                  &=frac12int_0^{infty}u^{-(p+1)/2}sum_{n=0}^{infty}frac{n!/(2n)!}{n!}(-u)^ndu
                  end{align}$$



                  By using the relation $Gamma(n)=(n-1)!$ which is valid for all $ninmathbb N$ we can consider the last integral as an application of Ramanujan's Master Theorem with $s=-frac{p-1}2$ and $phi(n)=frac{Gamma(n+1)}{Gamma(2n+1)}$. By finally using the Theorem we obtain



                  $$begin{align}
                  mathfrak{I}=frac12int_0^{infty}u^{-(p+1)/2}sum_{n=0}^{infty}frac{n!/(2n)!}{n!}(-u)^ndu&=frac12Gammaleft(-frac{p-1}2right)frac{Gammaleft(frac{p-1}2+1right)}{Gammaleft(2left(frac{p-1}2right)+1right)}\
                  &=frac1{2Gamma(p)}Gammaleft(1+frac{p-1}2right)Gammaleft(-frac{p-1}2right)
                  end{align}$$



                  Now by applying Euler's Reflection Formula with $z=1+frac{p-1}2$ we moreover get



                  $$begin{align}
                  mathfrak{I}=frac1{2Gamma(p)}Gammaleft(1+frac{p-1}2right)Gammaleft(-frac{p-1}2right)&=frac1{2Gamma(p)}frac{pi}{sinleft(pileft(1+frac{p-1}2right)right)}\
                  &=frac1{2Gamma(p)}frac{pi}{sinleft(frac{ppi}2+frac{pi}2right)}\
                  &=frac1{2Gamma(p)}frac{pi}{cosleft(frac{ppi}2right)}
                  end{align}$$



                  where within the last step the fundamental relation $sinleft(x+frac{pi}2right)=cos(x)$ was used. Thus for the original integral $mathfrak{I}$ we get




                  $$mathfrak{I}=int_0^{infty}cos(x)x^{-p}dx=frac{pi}{2Gamma(p)cosleft(pfrac{pi}2right)}$$








                  share|cite|improve this answer














                  share|cite|improve this answer



                  share|cite|improve this answer








                  edited Nov 24 at 22:23

























                  answered Nov 24 at 21:42









                  mrtaurho

                  2,5941827




                  2,5941827






















                      up vote
                      3
                      down vote













                      Hint: $displaystyleint_{0}^{infty}frac{cos x}{x^p}dx= text{Real part of}:int_{0}^{infty}frac{e^{iz}}{z^p}dz$ and use residue theorem. This has a pole of order $p$ hence the term $Gamma (p)$ in the denominator.






                      share|cite|improve this answer





















                      • This only works for integral $p$, right?
                        – AccidentalFourierTransform
                        Nov 24 at 22:13










                      • @AccidentalFourierTransform: Your comment makes me wonder whether there is a way to bootstrap from integer $p$ to the complex $p$...
                        – user21820
                        Nov 25 at 4:13










                      • @user21820 Something in the spirit of the Identity theorem, or more generally Analytic continuation, I guess...
                        – AccidentalFourierTransform
                        Nov 25 at 4:16










                      • @AccidentalFourierTransform: But here the answer is a meromorphic function of $p$, and we don't even have an accumulation point. So do you have a real proof? =)
                        – user21820
                        Nov 25 at 4:22










                      • @user21820 Oh, no, not at all. I was just pointing out that Yidati's answer was only valid for integer $p$. I didn't want to suggest that I have a better answer.
                        – AccidentalFourierTransform
                        Nov 25 at 4:26















                      up vote
                      3
                      down vote













                      Hint: $displaystyleint_{0}^{infty}frac{cos x}{x^p}dx= text{Real part of}:int_{0}^{infty}frac{e^{iz}}{z^p}dz$ and use residue theorem. This has a pole of order $p$ hence the term $Gamma (p)$ in the denominator.






                      share|cite|improve this answer





















                      • This only works for integral $p$, right?
                        – AccidentalFourierTransform
                        Nov 24 at 22:13










                      • @AccidentalFourierTransform: Your comment makes me wonder whether there is a way to bootstrap from integer $p$ to the complex $p$...
                        – user21820
                        Nov 25 at 4:13










                      • @user21820 Something in the spirit of the Identity theorem, or more generally Analytic continuation, I guess...
                        – AccidentalFourierTransform
                        Nov 25 at 4:16










                      • @AccidentalFourierTransform: But here the answer is a meromorphic function of $p$, and we don't even have an accumulation point. So do you have a real proof? =)
                        – user21820
                        Nov 25 at 4:22










                      • @user21820 Oh, no, not at all. I was just pointing out that Yidati's answer was only valid for integer $p$. I didn't want to suggest that I have a better answer.
                        – AccidentalFourierTransform
                        Nov 25 at 4:26













                      up vote
                      3
                      down vote










                      up vote
                      3
                      down vote









                      Hint: $displaystyleint_{0}^{infty}frac{cos x}{x^p}dx= text{Real part of}:int_{0}^{infty}frac{e^{iz}}{z^p}dz$ and use residue theorem. This has a pole of order $p$ hence the term $Gamma (p)$ in the denominator.






                      share|cite|improve this answer












                      Hint: $displaystyleint_{0}^{infty}frac{cos x}{x^p}dx= text{Real part of}:int_{0}^{infty}frac{e^{iz}}{z^p}dz$ and use residue theorem. This has a pole of order $p$ hence the term $Gamma (p)$ in the denominator.







                      share|cite|improve this answer












                      share|cite|improve this answer



                      share|cite|improve this answer










                      answered Nov 24 at 21:22









                      Yadati Kiran

                      1,282317




                      1,282317












                      • This only works for integral $p$, right?
                        – AccidentalFourierTransform
                        Nov 24 at 22:13










                      • @AccidentalFourierTransform: Your comment makes me wonder whether there is a way to bootstrap from integer $p$ to the complex $p$...
                        – user21820
                        Nov 25 at 4:13










                      • @user21820 Something in the spirit of the Identity theorem, or more generally Analytic continuation, I guess...
                        – AccidentalFourierTransform
                        Nov 25 at 4:16










                      • @AccidentalFourierTransform: But here the answer is a meromorphic function of $p$, and we don't even have an accumulation point. So do you have a real proof? =)
                        – user21820
                        Nov 25 at 4:22










                      • @user21820 Oh, no, not at all. I was just pointing out that Yidati's answer was only valid for integer $p$. I didn't want to suggest that I have a better answer.
                        – AccidentalFourierTransform
                        Nov 25 at 4:26


















                      • This only works for integral $p$, right?
                        – AccidentalFourierTransform
                        Nov 24 at 22:13










                      • @AccidentalFourierTransform: Your comment makes me wonder whether there is a way to bootstrap from integer $p$ to the complex $p$...
                        – user21820
                        Nov 25 at 4:13










                      • @user21820 Something in the spirit of the Identity theorem, or more generally Analytic continuation, I guess...
                        – AccidentalFourierTransform
                        Nov 25 at 4:16










                      • @AccidentalFourierTransform: But here the answer is a meromorphic function of $p$, and we don't even have an accumulation point. So do you have a real proof? =)
                        – user21820
                        Nov 25 at 4:22










                      • @user21820 Oh, no, not at all. I was just pointing out that Yidati's answer was only valid for integer $p$. I didn't want to suggest that I have a better answer.
                        – AccidentalFourierTransform
                        Nov 25 at 4:26
















                      This only works for integral $p$, right?
                      – AccidentalFourierTransform
                      Nov 24 at 22:13




                      This only works for integral $p$, right?
                      – AccidentalFourierTransform
                      Nov 24 at 22:13












                      @AccidentalFourierTransform: Your comment makes me wonder whether there is a way to bootstrap from integer $p$ to the complex $p$...
                      – user21820
                      Nov 25 at 4:13




                      @AccidentalFourierTransform: Your comment makes me wonder whether there is a way to bootstrap from integer $p$ to the complex $p$...
                      – user21820
                      Nov 25 at 4:13












                      @user21820 Something in the spirit of the Identity theorem, or more generally Analytic continuation, I guess...
                      – AccidentalFourierTransform
                      Nov 25 at 4:16




                      @user21820 Something in the spirit of the Identity theorem, or more generally Analytic continuation, I guess...
                      – AccidentalFourierTransform
                      Nov 25 at 4:16












                      @AccidentalFourierTransform: But here the answer is a meromorphic function of $p$, and we don't even have an accumulation point. So do you have a real proof? =)
                      – user21820
                      Nov 25 at 4:22




                      @AccidentalFourierTransform: But here the answer is a meromorphic function of $p$, and we don't even have an accumulation point. So do you have a real proof? =)
                      – user21820
                      Nov 25 at 4:22












                      @user21820 Oh, no, not at all. I was just pointing out that Yidati's answer was only valid for integer $p$. I didn't want to suggest that I have a better answer.
                      – AccidentalFourierTransform
                      Nov 25 at 4:26




                      @user21820 Oh, no, not at all. I was just pointing out that Yidati's answer was only valid for integer $p$. I didn't want to suggest that I have a better answer.
                      – AccidentalFourierTransform
                      Nov 25 at 4:26


















                      draft saved

                      draft discarded




















































                      Thanks for contributing an answer to Mathematics Stack Exchange!


                      • Please be sure to answer the question. Provide details and share your research!

                      But avoid



                      • Asking for help, clarification, or responding to other answers.

                      • Making statements based on opinion; back them up with references or personal experience.


                      Use MathJax to format equations. MathJax reference.


                      To learn more, see our tips on writing great answers.





                      Some of your past answers have not been well-received, and you're in danger of being blocked from answering.


                      Please pay close attention to the following guidance:


                      • Please be sure to answer the question. Provide details and share your research!

                      But avoid



                      • Asking for help, clarification, or responding to other answers.

                      • Making statements based on opinion; back them up with references or personal experience.


                      To learn more, see our tips on writing great answers.




                      draft saved


                      draft discarded














                      StackExchange.ready(
                      function () {
                      StackExchange.openid.initPostLogin('.new-post-login', 'https%3a%2f%2fmath.stackexchange.com%2fquestions%2f3012090%2fhow-to-evaluate-this-nonelementary-integral%23new-answer', 'question_page');
                      }
                      );

                      Post as a guest















                      Required, but never shown





















































                      Required, but never shown














                      Required, but never shown












                      Required, but never shown







                      Required, but never shown

































                      Required, but never shown














                      Required, but never shown












                      Required, but never shown







                      Required, but never shown







                      Popular posts from this blog

                      flock() on closed filehandle LOCK_FILE at /usr/bin/apt-mirror

                      Mangá

                       ⁒  ․,‪⁊‑⁙ ⁖, ⁇‒※‌, †,⁖‗‌⁝    ‾‸⁘,‖⁔⁣,⁂‾
”‑,‥–,‬ ,⁀‹⁋‴⁑ ‒ ,‴⁋”‼ ⁨,‷⁔„ ‰′,‐‚ ‥‡‎“‷⁃⁨⁅⁣,⁔
⁇‘⁔⁡⁏⁌⁡‿‶‏⁨ ⁣⁕⁖⁨⁩⁥‽⁀  ‴‬⁜‟ ⁃‣‧⁕‮ …‍⁨‴ ⁩,⁚⁖‫ ,‵ ⁀,‮⁝‣‣ ⁑  ⁂– ․, ‾‽ ‏⁁“⁗‸ ‾… ‹‡⁌⁎‸‘ ‡⁏⁌‪ ‵⁛ ‎⁨ ―⁦⁤⁄⁕